0 Daumen
616 Aufrufe

Aufgabe: $$\sum \limits_{k=0}^{n}{\frac{(-1)^{k}}{k+1}*\begin{pmatrix} n+1\\k \end{pmatrix}=\frac{1}{n+1}}$$

Im Screenshot könnt ihr meinen Fortschritt sehen.

An der letzten Stelle komme ich irgendwie nicht weiter, hat vielleicht einer eine Idee, oder habe ich vorher schon was falsch gemacht?

Mfg1.png

Avatar von

Hallo,

T hat zwar schon eine Lösung angegeben. Aber doch der Hinweis: Du hast den Induktionsschritt falsch begonnen: Dort muss an jeder Stelle der Formel, also auch im Binomialkoeffizienten n durch n+1 ersetzt werden. Das mach einen Induktionsbeweis sehr schwierig. Deshalb ist der Beweis von T auf jeden Fall vorzuziehen - wenn Du eine Wahl hast.

Gruß Mathhilf

1 Antwort

0 Daumen
 
Beste Antwort

Aloha :)

Wir wenden zuerst die Regel \(\binom{n+1}{k+1}=\frac{n+1}{k+1}\binom{n}{k}\) für den Binomialkoeffizienten an:$$\sum\limits_{k=0}^n\binom{n}{k}\frac{(-1)^k}{k+1}=\frac{1}{n+1}\sum\limits_{k=0}^n\binom{n}{k}\frac{n+1}{k+1}(-1)^k=\frac{1}{n+1}\sum\limits_{k=0}^n\binom{n+1}{k+1}(-1)^k$$

Jetzt machen wir eine Indexverschiebung, indem wir \(k\) von \(1\) bis \(n+1\) laufen lassen und dafür bei den Summanden \(k\) um \(1\) vermindern:$$=\frac{1}{n+1}\sum\limits_{k=1}^{n+1}\binom{n+1}{k}(-1)^{k-1}$$

Würde die Summe bei \(k=0\) beginnen, käme noch der Summand \(\binom{n+1}{0}(-1)^{0-1}=-1\) hinu. Das nutzen wir aus und lassen die Summe bei \(k=0\) beginnen, wobei die \(-1\) durch Addition einer \(1\) kompensiert wird:$$=\frac{1}{n+1}\left(\sum\limits_{k=0}^{n+1}\binom{n+1}{k}(-1)^{k-1}+1\right)=\frac{1}{n+1}\left(-\sum\limits_{k=0}^{n+1}\binom{n+1}{k}1^{(n+1)-k}(-1)^k+1\right)$$

Nach dem binomischen Lehrsatz ist \(\sum\limits_{k=0}^{n+1}\binom{n+1}{k}1^{(n+1)-k}(-1)^k=(1+(-1))^{n+1}=0\). Das heißt, die Summe fällt im Folgenden einfach weg:$$=\frac{1}{n+1}\left(-0+1\right)=\frac{1}{n+1}$$

Avatar von 148 k 🚀

Erstmal vielen Dank für die detaillierte und schnelle Antwort, mir ist leider jedoch ein Fehler unterlaufen.

Die Induktion lautet \(\sum\limits_{k=0}^n\frac{(-1)^k}{k+1}\binom{n}{k}=\frac{1}{n+1}\)

leider nicht \(\sum\limits_{k=0}^n\frac{(-1)^k}{k+1}\binom{n+1}{k}=\frac{1}{n+1}\),

kann es sein, dass du in deinem Beweis trotzdem mit \(\sum\limits_{k=0}^n\frac{(-1)^k}{k+1}\binom{n}{k}=\frac{1}{n+1}\) gerechnet hast, wie oben aufgeführt, wenn dem so wäre müsste doch am Ende \(\frac{1}{n+2}\) das Ergebnis sein oder nicht?

Mfg

Ja, ich habe schon vermutet, dass du dich mit \(\binom{n+1}{k}\) vertippt hast, weil das Ergebnis sonst nicht gepasst hätte. Daher habe ich mit \(\binom{n}{k}\) gerechnet.

Sorry, das hätte ich vielleicht dazu schreiben sollen.

Alles in Ordnung war ja mein Fehler, dennoch verstehe ich nicht, wieso ist das Ergebnis nicht \(\frac{1}{(n+1)+1}\) ist.

Ich habe den Ausdruck direkt umgeformt, vollständige Induktion war gar nicht nötig. Daher steht am Ende der vereinfachte Ausdruck \(\frac{1}{n+1}\).

Bei einer vollständigen Induktion baut man ja darauf auf, dass die Behauptung bereits für alle vorhergehenden \(n\) gezeigt ist. Ich habe das gar nicht benutzt, sondern den Ausdruck ohne vollständige Induktion direkt durch arithmetische Umformungen beweisen.

Ein anderes Problem?

Stell deine Frage

Willkommen bei der Mathelounge! Stell deine Frage einfach und kostenlos

x
Made by a lovely community